Exercise -1.46(New book) (BOMC-2011)

Discussion on Bangladesh National Math Camp
User avatar
sm.joty
Posts:327
Joined:Thu Aug 18, 2011 12:42 am
Location:Dhaka
Exercise -1.46(New book) (BOMC-2011)

Unread post by sm.joty » Mon Oct 31, 2011 5:58 pm

Let $x_1,x_2,.......x_n>0$ such that,\[\frac{1}{1+x_1}+\frac{1}{1+x_2}+\frac{1}{1+x_3}+\cdots\cdots+\frac{1}{1+x_n}=1 \]
Prove that , $x_1x_2.........x_n\geq(n-1)^{n}$
হার জিত চিরদিন থাকবেই
তবুও এগিয়ে যেতে হবে.........
বাধা-বিঘ্ন না পেরিয়ে
বড় হয়েছে কে কবে.........

sourav das
Posts:461
Joined:Wed Dec 15, 2010 10:05 am
Location:Dhaka
Contact:

Re: Exercise -1.46(New book) (BOMC-2011)

Unread post by sourav das » Mon Oct 31, 2011 6:07 pm

At first tell us about your work and where do you stuck
You spin my head right round right round,
When you go down, when you go down down......
(-$from$ "$THE$ $UGLY$ $TRUTH$" )

User avatar
sm.joty
Posts:327
Joined:Thu Aug 18, 2011 12:42 am
Location:Dhaka

Re: Exercise -1.46(New book) (BOMC-2011)

Unread post by sm.joty » Mon Oct 31, 2011 6:50 pm

আমার একটা সমাধান আছে । কিন্তু মনে হচ্ছে সমাধানের শেষ কয়েক লাইনে মনে হয় কিছু একটা ভুল আছে। দেখুনতো কি মনে হয়। :geek:
Here, $x_1,x_2,........x_n>0$ such that $\frac{1}{1+x_1}+\frac{1}{1+x_2}+\cdots\cdots+\frac{1}{1+x_n}=1$
Let $a_1=1+x_1, a_2=1+x_2,.................a_n=1+x_n$
so we can write,$\frac{1}{a_1}+\frac{1}{a_2}+\cdots\cdots+\frac{1}{a_n}=1$
$\Rightarrow \frac{n}{\frac{1}{a_1}+\frac{1}{a_2}+\cdots\cdots+\frac{1}{a_n}}=n$
Now using AM-GM-HM inequality,
$\Rightarrow \frac{n}{\frac{1}{a_1}+\frac{1}{a_2}+\cdots\cdots+\frac{1}{a_n}}\leq\sqrt[n]{a_1a_2a_3......a_n}\leq\frac{a_1+a_2+a_3+\cdots\cdots+a_n}{n}$
that means
\[a_1+a_2+\cdots\cdots+a_n\geq n^2\]
\[\Rightarrow (1+x_1)+(1+x_2)+\cdots\cdots+(1+x_n) \geq n^2\]
\[\Rightarrow n+(x_1+x_2+\cdots\cdots+x_n) \geq n^2\]
\[\Rightarrow x_1+x_2+\cdots\cdots+x_n \geq n(n-1)\]
\[\Rightarrow \frac{x_1+x_2+\cdots\cdots+x_n}{n} \geq n-1\]
$\Rightarrow AM \geq n-1$ where AM is the arithmetic mean. So the geometric mean is GM
\[GM=\sqrt[n]{x_1x_2.......x_n}\] And $AM \geq GM$
So, \[\frac{AM}{AM}\geq \frac{n-1}{GM}\]
$\Rightarrow GM\geq n-1$
So,\[x_1x_2x_3.............x_n \geq (n-1)^{n}\]
হার জিত চিরদিন থাকবেই
তবুও এগিয়ে যেতে হবে.........
বাধা-বিঘ্ন না পেরিয়ে
বড় হয়েছে কে কবে.........

sourav das
Posts:461
Joined:Wed Dec 15, 2010 10:05 am
Location:Dhaka
Contact:

Re: Exercise -1.46(New book) (BOMC-2011)

Unread post by sourav das » Mon Oct 31, 2011 7:25 pm

$a \geq b$ then $\frac {1}{a} \leq \frac {1}{b}$
Then tell me where is the bug in your solution?
You spin my head right round right round,
When you go down, when you go down down......
(-$from$ "$THE$ $UGLY$ $TRUTH$" )

User avatar
*Mahi*
Posts:1175
Joined:Wed Dec 29, 2010 12:46 pm
Location:23.786228,90.354974
Contact:

Re: Exercise -1.46(New book) (BOMC-2011)

Unread post by *Mahi* » Mon Oct 31, 2011 7:28 pm

sm.joty wrote: $\Rightarrow AM \geq n-1$... And $AM \geq GM$
So, \[\frac{AM}{AM}\geq \frac{n-1}{GM}\]
$\Rightarrow GM\geq n-1$
You can't divide inequalities.
Please read Forum Guide and Rules before you post.

Use $L^AT_EX$, It makes our work a lot easier!

Nur Muhammad Shafiullah | Mahi

Post Reply